Let $a, b, c$ be positive reals such that $abc=1$. Show that \[\frac{1}{a^5(b+2c)^2} + \frac{1}{b^5(c+2a)^2} + \frac{1}{c^5(a+2b)^2} \ge \frac{1}{3}.\]
Problem
Source:
Tags: inequalities, Hi
26.07.2010 19:24
MellowMelon wrote: Let $a, b, c$ be positive reals such that $abc=1$. Show that \[\frac{1}{a^5(b+2c)^2} + \frac{1}{b^5(c+2a)^2} + \frac{1}{c^5(a+2b)^2} \ge \frac{1}{3}.\] replacing $a,b,c$ by $\frac{1}{a},\frac{1}{b},\frac{1}{c}$ yields that the inequality is equivalent to $ \sum \frac{a^5b^2c^2}{(c+2b)^2} \ge \frac{1}{3}$ or $\sum \frac{a^3}{(c+2b)^2} \ge \frac{1}{3} $ which is just Holder inequality because $\sum \frac{a^3}{(c+2b)^2}\sum (2b+c)\sum(2b+c) \ge (a+b+c)^3$ and $a+b+c \ge 3$
26.07.2010 19:26
Let $a=x^6, b=y^6, c=z^6$ so $xyz=1$. Then it suffices to show \begin{align*} \sum\frac{1}{a^5(b+2c)^2} = \sum\frac{y^{30}z^{30}}{(y^6+2z^6)^2} &\ge \frac{(\sum{y^{15}z^{15}})^2}{\sum(y^6+2z^6)^2}\\ &= \frac{\sum{y^{30}z^{30}}+2\sum{x^{30}y^{15}z^{15}}}{5\sum{x^{12}}+4\sum{y^6z^6}}\ge\frac13,\end{align*}or, after homogenizing, \[3\sum{y^{30}z^{30}}+6\sum{x^{30}y^{15}z^{15}} \ge 5\sum{x^{28}y^{16}z^{16}}+4\sum{x^{16}y^{22}z^{22}},\]which is just Muirhead.
26.07.2010 19:30
MellowMelon wrote: Let $a, b, c$ be positive reals such that $abc=1$. Show that \[\frac{1}{a^5(b+2c)^2} + \frac{1}{b^5(c+2a)^2} + \frac{1}{c^5(a+2b)^2} \ge \frac{1}{3}.\] Is this problem in Mathematical Reflections? Let $(x,y,z)=(a^{-1}, b{-1}, c^{-1})^$. Then the inequality to prove is $\sum \frac{x^3}{(2y+z)^2}\ge \frac{1}{3}$ Using Jensen's inequality applied to $f(t)=t^{-2}$, we have that $\sum xf(\frac{2y+z}{x})\ge (x+y+z)f(3)=\frac{x+y+z}{9}$ But the last one is greater than $\frac {1}{3}$ because $x+y+z\ge 3$
26.07.2010 19:32
Titu Andreescu is one of the people in charge of Mathematical Reflections, isn't he? He's the one who proposed the problem, so it wouldn't surprise me if it appeared there too (although probably after the fact).
26.07.2010 19:34
MellowMelon wrote: Titu Andreescu is one of the people in charge of Mathematical Reflections, isn't he? He's the one who proposed the problem, so it wouldn't surprised me if it appeared there too (although probably after the fact). Yes =) This problem is very similar to IMO 95/2
26.07.2010 19:50
MellowMelon wrote: Let $a, b, c$ be positive reals such that $abc=1$. Show that \[\frac{1}{a^5(b+2c)^2} + \frac{1}{b^5(c+2a)^2} + \frac{1}{c^5(a+2b)^2} \ge \frac{1}{3}.\] Another solution: By Cauchy-Schwarz ineq, we have: $\sum\limits_{cyc} {\frac{1}{{{a^5}{{(b + 2c)}^2}}}} = \sum\limits_{cyc} {\frac{{{b^4}{c^4}}}{{a{{(b + 2c)}^2}}}} \ge \frac{{{{({a^2}{b^2} + {b^2}{c^2} + {c^2}{a^2})}^2}}}{{a{{(b + 2c)}^2} + b{{(c + 2a)}^2} + c{{(a + 2b)}^2}}}$ we need to prove that: $\frac{{{{({a^2}{b^2} + {b^2}{c^2} + {c^2}{a^2})}^2}}}{{a{{(b + 2c)}^2} + b{{(c + 2a)}^2} + c{{(a + 2b)}^2}}} \ge \frac{1}{3}$ $\Leftrightarrow 3({a^4}{b^4} + {b^4}{c^4} + {c^4}{a^4} + 2({a^2} + {b^2} + {c^2})) \ge \sum\limits_{cyc} a {b^2} + 4\sum\limits_{cyc} {{a^2}b} + 12$ By AM-GM ineq, we have: $RHS \le \frac{5}{2}({a^2}{b^2} + {b^2}{c^2} + {c^2}{a^2} + {a^2} + {b^2} + {c^2}) + 12$ And $LHS \ge 6({a^2}{b^2} + {b^2}{c^2} + {c^2}{a^2}) + 6({a^2} + {b^2} + {c^2}) - 9$ $\ge \frac{5}{2}({a^2}{b^2} + {b^2}{c^2} + {c^2}{a^2} + {a^2} + {b^2} + {c^2}) + 12$ ( because abc=1) So we have done .
26.07.2010 22:09
For $x=\frac{1}{a}, y=\frac{1}{b}, z=\frac{1}{c},$ we get \[LHS=\frac{1}{a^{5}(b+2c)^{2}}+\frac{1}{b^{5}(c+2a)^{2}}+\frac{1}{c^{5}(a+2b)^{2}}=(xyz)^2\cdot[\frac{x^3}{(2y+z)^2}+\frac{y^3}{(2z+x)^2}+\frac{z^3}{(2x+y)^2}]\stackrel{(xyz=1)}{=}\] \[=\frac{x^3}{(2y+z)^2}+\frac{y^3}{(2z+x)^2}+\frac{z^3}{(2x+y)^2}\stackrel{(Holder's\ inequality)}{\geq}\frac{(x+y+z)^3}{(2y+z+2z+x+2x+y)^2}=\frac{x+y+z}{9}\stackrel{(AM-GM)}{\geq}\] \[\geq\frac{\sqrt[3]{xyz}}{3}\stackrel{(xyz=1)}{=}\frac{1}{3}=RHS\] IMPORTANT REMARK Have a look at my old problem here. You'll see that my problem is a generalization of IMO 1995 inequality and for this one (which is O161, from Mathematical Reflections 3/2010, proposed by Titu Andreescu).
03.08.2010 18:02
By am-gm : $\frac{(bc)^3}{(ab+2ac)^2}+\frac{ab+2ac}{27}+\frac{ab+2ac}{27} \ge \frac{bc}{3}$ $\Leftrightarrow $ $LHS \ge \frac{ab+bc+ac}{9} \ge \frac{3\sqrt[3]{(abc)^2}}{9}=1/3$
04.08.2010 11:40
MellowMelon wrote: Let $a, b, c$ be positive reals such that $abc=1$. Show that \[\frac{1}{a^5(b+2c)^2} + \frac{1}{b^5(c+2a)^2} + \frac{1}{c^5(a+2b)^2} \ge \frac{1}{3}.\] Put $a=\frac{1}{x}, b=\frac{1}{y}, c=\frac{1}{z}$ then: $x,y,z >0, xyz=1$ $\frac{1}{a^5(b+2c)^2}+\frac{1}{b^5(c+2a)^2} + \frac{1}{c^5(a+2b)^2} \ge \frac{1}{3}$ $\Leftrightarrow \sum\frac{x^3}{(2y+z)^2}\ge \frac{1}{3}$ By AG-GM: $ \sum[\frac{x^3}{(2y+z)^2}+\frac{2y+z}{27}+\frac{2y+z}{27}]\ge \sum[\frac{x}{3}]$ $\Leftrightarrow \sum\frac{x^3}{(2y+z)^2}\ge \frac{1}{9}(x+y+z)\ge \frac{1}{3}$ equati if only if $a=b=c=1$
07.09.2010 13:41
I just recently thought about this problem, so I figured I post my solution (which doesn't involve Holder). Note that if we do the substitutions $a\to \frac 1x$ and so on, the inequality becomes \[ \sum \frac {x^3}{(2y+z)^2} \geq \frac 13 . \] A natural idea is to try Cauchy-Schwarz, so we do so: \[ \sum \frac {x^3}{(2y+z)^2} \cdot \sum x\geq \left( \sum \frac {x^2}{2y+z} \right)^2 \] However, we know that we can apply Cauchy-Schwarz for the sum in the RHS above, so \[ \sum \frac {x^2}{2y+z} \cdot \sum (2y+z) \geq \left( \sum x \right)^2 \ \Rightarrow \ \sum \frac {x^2}{2y+z} \geq \frac 13 \sum x . \] Putting all this together we get \[ \sum \frac {x^3}{(2y+z)^2} \geq \frac 19 \sum x \geq \frac 19 \cdot 3 = \frac 13 , \] via AM-GM because $xyz=1$.
11.04.2013 17:21
Note $\sum \frac {1}{a^5(b+2c)^2}=\sum\frac {(bc)^3}{(ab+2ac)^2}$. Now, $(\sum\frac {(bc)^3}{(ab+2ac)^2})(\sum \frac {1}{bc})\geq (\sum\frac {bc}{ab+2ac})^2$. Also, $(\sum\frac {bc}{ab+2ac})(\sum \frac {bc(2c+b)}{bc})\geq \frac{\sum(bc)^2}{3(a+b+c)}$. So now it's required to show $(\sum ab)^4\geq 9(\sum a)^2$, which is obvious since, $(\sum ab)^2\geq 3\sum a^2bc=3\sum a$ , so done.
17.09.2013 10:26
Generalization of USA TST 2010 Generalization of USA TST 2010(2)
01.12.2013 21:15
Hölder kills it directly. \[ { \left(\sum_{cyc} a(b+2c) \right) \left(\sum_{cyc} a(b+2c) \right) \left(\sum_{cyc} \frac{1}{a^5(b+2c)^2} \right) \geq \left(\sum_{cyc}\frac{1}{a}\right)^3 =(ab+bc+ac)^3 } \] which is equivalent to \[ \left(\sum_{cyc} \frac{1}{a^5(b+2c)^2} \right) \geq \frac{ab+bc+ca}{9} \] and by AM-GM we are done.
08.01.2015 22:55
Partially clear denominators and simplify; we want to show that \[ Q=\sum_{cyc}\frac{b^5c^5}{(b+2c)^2}\ge \frac{1}{3}.\] But by Holder, $(Q)\left( \sum_{cyc} \frac{b+2c}{bc}\right) ^2\ge (ab+bc+ca)^3.$ Since $\frac{b+2c}{bc}=ab+2ac$, we have $Q\ge \frac{ab+bc+ca}{9}\ge \frac{\sqrt[3]{(abc)^2}}{9}=\frac{1}{3}$.
04.03.2015 20:00
$a=\frac{vw}{u^2}$, $b=\frac{wu}{v^2}$, $c=\frac{uv}{w^2}$. Assume WLOG that $u^9+v^9+w^9=1$. Inequality to prove is \[\sum\frac{u^9}{uvw(w^3+2v^3)^2}=\sum u^9f(u^3-v^3)\ge\frac13\] where $f(x)=\frac{1}{uvw(u^3+v^3+w^3-x)^2}$ is convex and increasing over the domain $u^9+v^9+w^9=1$. Thus, \[\sum u^9f(u^3-v^3)\ge f(\underbrace{u^{12}+v^{12}+w^{12}-u^9v^3-v^9w^3-w^9u^3}_{\ge0})\ge f(0)=\frac1{uvw(u^3+v^3+w^3)^2}\ge\frac13\] by AM-GM
12.04.2015 20:14
my solution = let $S=\frac{1}{a^5(b+2c)^2} + \frac{1}{b^5(c+2a)^2} + \frac{1}{c^5(a+2b)^2}$ than by holder's we have $(S^{1/3})(a(b+2c)+b(c+2a)+c(a+2b))^{2/3} \ge \frac{1}{a}+\frac{1}{b}+\frac{1}{c}=ab+bc+ca$ which is equivalent to $S(3(ab+bc+ca))^2\ge (ab+bc+ca)^3$ thus it remains to prove that $ab+bc+ca\ge 3$ which is trivial by AM-GM as $ab+bc+ca\ge 3(abc){2/3} = 3$ as $abc=1$ so we are done
28.05.2015 22:11
$\displaystyle\sum_{\text{cyc}} \displaystyle\frac{1}{a^5(b+2c)^2} =\displaystyle\sum_{\text{cyc}} \displaystyle\frac{(1/a)^3}{((1/c)+(2/b))^2} \ge \displaystyle\frac{\left( \displaystyle\sum_{\text{cyc}} (1/a) \right)^3}{\left( \displaystyle\sum_{\text{cyc}} (1/c)+(2/b) \right)^2} = \displaystyle\frac{T^3}{(3T)^2} = \displaystyle\frac{T}{9} \ge 1/3$ with $T=ab+bc+ca \ge 3$. By Holder in fraction form and $p=3,q=3/2$
03.08.2015 04:03
A not-so-elegant solution: First, we try to simplify the denominators on the LHS. We have, \[\frac{1}{a^5(b+2c)^2}=\frac{1}{a^5(b^2+4bc+4c^2)}\geq\frac{1}{a^5(3b^2+6c^2)},\] so it is enough to show that \[\sum_{cyc}\frac{1}{a^5(b^2+2c^2)}\geq1.\] We know that, by $abc=1$, we have \[\sum_{cyc}\frac{1}{a^5(b^2+2c^2)}=\sum_{cyc}\frac{b^6c^6}{bc(b^2+2c^2)}=\sum_{cyc}\frac{(b^3c^3)^2}{b^3c+2bc^3},\] so we use Titu on the LHS after which it is enough to show that \[\frac{(a^3b^3+b^3c^3+c^3a^3)^2}{\sum_{cyc}(a^3b+2ab^3)}\geq1.\] By weighted AM-GM, we have \[\frac{1}{9}a^6b^6+\frac{1}{9}a^3b^6c^3+\frac{7}{9}a^6b^3c^3\geq a^{\frac{17}{3}}b^{\frac{11}{3}}c^{\frac{8}{3}}=a^3b,\] so \[\frac{1}{9}\sum_{cyc}a^6b^6+\frac{8}{9}\sum_{cyc}a^6b^3c^3\geq\sum_{cyc}a^3b.\] Similarly, \[\frac{2}{9}\sum_{cyc}a^6b^6+\frac{16}{9}\sum_{cyc}a^6b^3c^3\geq2\sum_{cyc}ab^3,\] so \[\frac{1}{3}\sum_{cyc}a^6b^6+\frac{8}{3}\sum_{cyc}a^6b^3c^3\geq\sum_{cyc}(a^3b+2ab^3).\] However, the LHS is obviously at most $\sum_{cyc}a^6b^6+2\sum_{cyc}a^6b^3c^3$, so \[\sum_{cyc}a^6b^6+2\sum_{cyc}a^6b^3c^3\geq\sum_{cyc}(a^3b+2ab^3).\] The LHS above is the square of $a^3b^3+b^3c^3+c^3a^3$ and the RHS is positive, so we have \[\frac{(a^3b^3+b^3c^3+c^3a^3)^2}{\sum_{cyc}(a^3b+2ab^3)}\geq1,\] and we are done.
18.12.2015 17:58
My solution, which is very similar to aditya21's: By Holder's inequality, $(\sum_{cyc}a(b+2c))^{\frac{2}{3}} (\sum_{cyc}\frac{1}{a^5(b+2c)^2})^{\frac{1}{3}} \ge \sum_{cyc} \frac{1}{a} = ab+bc+ca$. Thus, $\sum_{cyc}\frac{1}{a^5(b+2c)^2} \ge \frac{ab+bc+ca}{9}$. By AM-GM, $\frac{ab+bc+ca}{9} \ge \frac{1}{3}$, as requested.
20.08.2017 16:33
Replace $a, b, c$ with $\frac{1}{x}, \frac{1}{y}, \frac{1}{z}$, respectively; the inequality becomes $\sum_{\text{cyc}}\frac{x^3}{(z+2y)^2}\geq \frac{1}{3}.$
Note that from AM-GM we have $x+y+z\geq 3\sqrt[3]{xyz}=3$, so from our lemma we have \[\sum_{\text{cyc}}\frac{x^3}{(z+2y)^2}\geq \frac{(x+y+z)^3}{(3x+3y+3z)^2}=\frac{x+y+z}{9}\geq \frac{1}{3},\]as desired.
20.08.2017 17:29
AlgebraFC wrote: Replace $a, b, c$ with $\frac{1}{x}, \frac{1}{y}, \frac{1}{z}$, respectively; the inequality becomes $\sum_{\text{cyc}}\frac{x^3}{(z+2y)^2}\geq \frac{1}{3}.$
Note that from AM-GM we have $x+y+z\geq 3\sqrt[3]{xyz}=3$, so from our lemma we have \[\sum_{\text{cyc}}\frac{x^3}{(z+2y)^2}\geq \frac{(x+y+z)^3}{(3x+3y+3z)^2}=\frac{x+y+z}{9}\geq \frac{1}{3},\]as desired. The Lemma is a direct consequence of Holder.
25.02.2021 20:51
posting the solution JFF \[f(x)=\frac{1}{x^2} \quad \textit{the second derivative is positive so it is convex}\]\[x=1/a,y=1/b=z=1/c\]\[\sum \frac{x^3}{(z+2y)^2}=\sum xf(\frac{z+2y}{x^2}) \ge (\sum x)f(3) \ge \frac13\]\[\iff x+y+z \ge 3\sqrt[3]{xyz} =3\]
11.07.2021 12:17
17.04.2022 09:03
By Holder, $$\left(\sum_{\text{cyc}}\frac{1}{a^5(b+2c)^2}\right)\left(\sum_{\text{cyc}}a(b+2c)\right)^2\ge\left(\sum_{\text{cyc}}\frac{1}{a}\right)^3,$$so $$\sum_{\text{cyc}}\frac{1}{a^5(b+2c)^2}\ge\frac{(ab+bc+ca)^3}{(3ab+3bc+3ca)^2}\ge\frac{1}{3}$$by AM-GM. $\square$
05.01.2023 04:33
We multiply by $(abc)^5$ to get $$\sum_{cyc} \frac{b^5c^5}{(b+2c)^2}\geq \frac{1}{3}.$$We will then homogenize: $$\sum_{cyc}\frac{b^5c^5}{(abc)^\frac{8}{3}(b+2c)^2}\geq\frac{1}{3}$$Due to the condition $abc=1$, since this is homogenous, we can 6th power each variable (essentially the substitution $a\rightarrow a'^6$ etc.) so that this becomes $$\sum_{cyc}\frac{b^{30}c^{30}}{(abc)^{16}(b^6+2c^6)^2}\geq\frac{1}{3}$$We can then expand: $$\sum_{cyc}\frac{b^{30}c^{30}}{a^{28}b^{16}c^{16}+4a^{16}b^{22}c^{22}+4a^{16}b^{16}c^{28}}\geq\frac{1}{3}$$By Titu's, we have $$\sum_{cyc}\frac{b^{30}c^{30}}{a^{28}b^{16}c^{16}+4a^{16}b^{22}c^{22}+4a^{16}b^{16}c^{28}}\geq \frac{(a^{15}b^{15}+b^{15}c^{15}+c^{15}a^{15})^2}{5cyc(28,16,16)+4cyc(22,22,16)}.$$Therefore, it remains to show that $$\frac{(a^{15}b^{15}+b^{15}c^{15}+c^{15}a^{15})^2}{5cyc(28,16,16)+4cyc(22,22,16)}\geq\frac{1}{3},$$which becomes $$3cyc(30,30,0)+6cyc(30,15,15)\geq 5cyc(28,16,16)+4cyc(22,22,16),$$which follows by Muirhead's.
16.03.2023 23:41
It is equivalent to show that $$\sum_{\mathrm{cyc}} \frac{b^5c^5}{(b+2c)^2} \geq \frac{1}{3},$$or after applying Cauchy-Schwarz, $$(\sqrt{a^5b^5}+\sqrt{b^5c^5}+\sqrt{c^5a^5})^2\geq \frac{5}{3}(a^2+b^2+c^2)+\frac{4}{3}(ab+bc+ca) \iff 3[5,5,0]+6[5,\tfrac{5}{2},\tfrac{5}{2}] \geq 5[\tfrac{14}{3},\tfrac{8}{3},\tfrac{8}{3}]+4[\tfrac{11}{3},\tfrac{11}{3},\tfrac{8}{3}].$$Since $[5,\tfrac{5}{2},\tfrac{5}{2}]\geq [\tfrac{11}{3},\tfrac{11}{3},\tfrac{8}{3}]$ by Muirhead, it suffices to show that $$3[5,5,0]+2[5,\tfrac{5}{2},\tfrac{5}{2}]\geq 5[\tfrac{14}{3},\tfrac{8}{3},\tfrac{8}{3}].$$(Weighted) AM-GM on the LHS gives $3[5,5,0]+2[5,\tfrac{5}{2},\tfrac{5}{2}]\geq 5[5,4,1]$, and Muirhead finishes. $\blacksquare$
30.04.2023 23:59
Rewrite $\frac 1{a^5(b+2c)^2} = \frac{b^3c^3}{a^2(b+2c)^2}.$ Now Holder in the form $$\left(\sum \frac{b^3c^3}{a^2(b+2c)^2}\right) \left(\sum a(b+2c)\right)^2 \geq \frac 13 (ab+bc+ca)^3$$suffices as $ab+bc+ca \geq 3$.
25.08.2023 22:20
Substitute $a = \frac{x}{y}, b = \frac{y}{z}, c = \frac{z}{x}$. This becomes by Cauchy-Schwarz, \[ \left(\sum_{\text{cyc}} \frac{y^5z^2}{x^3(xy + 2z^2)^2}\right)\left(\sum_{\text{cyc}} \frac{x}{y}\right) \ge \left(\sum_{\text{cyc}} \frac{y^2z}{x(xy + 2z)}\right)^2 \] It remains to show that \[ \sum_{\text{cyc}} \frac{y^2z}{x(xy + 2z)} \ge 1 \] Since by C-S again it follows that \[ \left(\sum_{\text{cyc}} \frac{y^2z}{x(xy + 2z)}\right)\left(\sum_{\text{cyc}} xz(xy + 2z)\right) \ge (\sum_{\text{cyc}} yz)^2 \]this is just \[ \sum_{\text{cyc}} x^2yz + 2xz^2 \le \sum_{\text{cyc}} y^2z^2 + 2x^2yz \]which follows by muirhead's.
29.10.2023 22:19
We substitute $(a,b,c) = \left(\frac 1x, \frac 1y, \frac 1z\right)$, where $x,y,z$ are positive reals and $xyz=1$. Then it suffices to show \[\sum_{\text{cyc}} \frac{1}{a^5(b+2c)^2} = \sum_{\text{cyc}} \frac{x^5}{\left(\frac 1y + \frac 2z\right)^2} = \sum_{\text{cyc}} \frac{x^5y^2z^2}{(2y+z)^2} = \sum_{\text{cyc}} \frac{x^3}{(2y+z)^2} \ge \frac 13.\] Note that Holder's gives us \[\sum_{\text{cyc}} \frac{x^3}{(2y+z)^2} \sum_{\text{cyc}} (2y+z) \sum_{\text{cyc}} (2y+z) \ge \left(\sum_{\text{cyc}} x\right)^3.\] Simplifying and using AM-GM gives \begin{align*} \sum_{\text{cyc}} \frac{x^3}{(2y+z)^2} &\ge \frac{(x+y+z)^3}{9(x+y+z)^2} \\ &\ge \frac 19 (x+y+z) \\ &\ge \frac 19 (3 \sqrt[3]{xyz}) \\ &\ge \frac 13.~\blacksquare \end{align*}
30.10.2023 22:00
There was gen 1 here.
30.10.2023 22:01
There was gen 2 here.
22.11.2023 07:52
We first multiply the LHS$=X$ by $(abc)^3$, getting: \[X=\sum_{cyc}{\frac{b^3c^3}{a^2(b+2c)^2}}.\]Applying Holders, we get: \[X\cdot \left(\sum_{cyc}{a(b+2c)}\right)^2\geq (ab+bc+ca)^3,\]\[X\geq \frac{1}{9}(ab+bc+ca).\]It suffices to show $ab+bc+ca\geq 3$, which is true by AM-GM, as desired.
24.11.2023 20:35
Consider applying Holders to find, \begin{align*} \left( \sum_{cyc} \frac{1}{a^5(b+2c)^2} \right) \left( \sum_{cyc} a(b+2c) \right)^2 \geq \left( \sum_{cyc} \frac{1}{a} \right)^3 = (ab+bc+ca)^3 \end{align*}Then we wish to show that, \begin{align*} \frac{(ab+bc+ca)^3}{\left(3ab+3bc+3ca \right)^2} \geq \frac{1}{3} \end{align*}Which simplifies to showing, \begin{align*} \frac{ab+bc+ca}{9} \geq \frac13 \end{align*}But AM-GM gives, \begin{align*} ab + bc + ca \geq 3 \end{align*}so we are done.
01.01.2024 03:03
bÿ Holdër's änd äm-gm wë hävë \[\left(\frac{1}{a^5(b+2c)^2} + \frac{1}{b^5(c+2a)^2} + \frac{1}{c^5(a+2b)^2}\right)(a(b+2c)+b(c+2a)+c(a+2b))^2=\left(\frac{1}{a^5(b+2c)^2} + \frac{1}{b^5(c+2a)^2} + \frac{1}{c^5(a+2b)^2}\right)9(ab+bc+ca)^2 \ge \left(\frac 1a+\frac 1b+\frac 1c\right)^3=(ab+bc+ca)^3\ge3(ab+bc+ca)^2\]
25.02.2024 20:36
By Holder we have: $$[\sum_{cyc}{\frac{1}{a^5(b+2c)^2}}]^{\frac{1}{3}}[\sum_{cyc}{a(b+2c)}]^{\frac{1}{3}}[\sum_{cyc}{a(b+2c)}]^{\frac{1}{3}}\geq [\sum_{cyc}{\frac{1}{a}}]$$And we can finish with AM-GM: $$\sum_{cyc}{\frac{1}{a^5(b+2c)^2}}\geq \frac{ab+bc+ca}{9}\geq \frac{1}{3}$$
31.07.2024 21:45
Note that, by Holder's, \begin{align*} \sum_{\text{cyc}} \dfrac{1}{a^5\left(b+2c\right)^2} = \sum_{\text{cyc}} \dfrac{a^3b^3}{(ca + 2bc)^2} &\geq \dfrac{\displaystyle \sum_{\text{cyc}} \left(\dfrac{\left(a^3b^3\right)^{1/3}}{(ca+2bc)^{2/3}} \cdot \left(ca+2bc\right)^2\right)^3}{\left(\displaystyle \sum_{\text{cyc}}\left(ca+2bc\right)\right)^2}. \end{align*}This is equivalent to \begin{align*} \dfrac{\left(\displaystyle \sum_{\text{cyc}} ab\right)^3}{\left(\displaystyle \sum_{\text{cyc}}\left(ca+2bc\right)\right)^2} \geq \dfrac{(ab+ca+ca)^3}{\left(3(ab+bc+ca)\right)^2} = \dfrac{ab+bc+ca}{9} \geq \dfrac{1}{3}. \end{align*}The second inequality comes from AM-GM.
03.08.2024 06:40
By applying Holders, we get that \[\left(\sum_{\text{cyc}} \frac{1}{a^5(b+2c)^2}\right)\left(\sum_{\text{cyc}} b+2c\right)^2 \left(\sum_{\text{cyc}} a \right) \ge \left(\sum_{\text{cyc}} a\right)^4\]\[\implies \sum_{\text{cyc}} \frac{1}{a^5(b+2c)^2} \ge \frac{a+b+c}{9} \ge \frac{1}{3}\]where the last inequality comes from AM-GM since $abc = 1$.
06.10.2024 01:03
I have discussed this problem in my holder video on the inequalities playlist on my youtube channel "little fermat" . Here is the Link
13.01.2025 17:17
By Holder, we have $(\sum_{cyc} \frac{1}{a^5(b+2c)^2}) (\sum_{cyc} a(b + 2c))^2 \ge (\sum_{cyc} \frac 1a)^3$, noting that $\sum_{cyc} a(b + 2c) = 3ab + 3bc + 3ac = 3(\frac 1a + \frac 1b + \frac 1c)$, it is sufficient to prove that $\frac{1}{a} + \frac 1b + \frac 1c \ge 3$, which is obvious by AM-GM.